0 Daumen
524 Aufrufe

Welche der folgenden Aussagen sind fur beliebige Endomorphismen f ∈ Hom(Rn;Rn)
mit Matrix A (kanonische Basis) wahr? Geben Sie bei positiven Antworten eine kurze
Begrünndung und bei negativen Antworten ein Gegenbeispiel:


a) Ist f eine Isometrie, dann gilt jdet(A)j = 1.


b) Ist jdet(A)j = 1, dann ist f eine Isometrie.

c) Ist f eine Isometrie und ein selbstadjungierter Endomorphismus, dann hat f einen
Eigenwert λ≠ 0.

d) Ist f ein selbstadjungierter Endomorphismus und ist det(A) ≠ 0, dann hat f einen
Eigenwert λ≠ 0.

e) Ist f ein selbstadjungierter Endomorphismus und hat f einen Eigenwert λ≠ 0. dann
ist det(A) ≠ 0

Avatar von

a) Ist f eine Isometrie, dann gilt jdet(A)j = 1.

Diese j hier sind Betragsstriche?

ja,das sind betragsstriche

Ein anderes Problem?

Stell deine Frage

Willkommen bei der Mathelounge! Stell deine Frage einfach und kostenlos

x
Made by a lovely community